lukemhearn
Thanks Received: 0
Vinny Gambini
Vinny Gambini
 
Posts: 8
Joined: March 02nd, 2009
 
 
 

PT33, S4, G2 - Birds in a Forest

by lukemhearn Fri Jun 05, 2009 10:22 pm

I can't, for the life of me, figure out why the answers to questions 10-12.

I think I've figured out why the answer to 11 is A (because if G is in, H is out, which means J and M are out because of the second condition, which, if J is out, S must be in), but that second condition does give me a slight headache, so I can't tell if I'm reading it right (based on 11, anyway), or if I'm just inventing a chain of logic that works.

Thanks.
User avatar
 
noah
Thanks Received: 1192
Atticus Finch
Atticus Finch
 
Posts: 1541
Joined: February 11th, 2009
 
 
 

Re: E33, S4, P10-12

by noah Tue Jun 09, 2009 10:52 am

Preptest 33 December 2000 LSAT Answers

Make sure you drew your diagram correctly (see attached).

For #10, when J is in, W must be out.

For #11, you've got it correct.

For #12, you need to add an arrow from H in to S out and from S in to H out. Each of the four other choices can't be true:
(A) S in leads to H, J and M out.
(C) S in leads to H, J and M out.
(D) J out leads to S in, to H, (J) and M out.
(E) There's no combination of four birds in that can work. So many birds force others out!

Does that clear it up?
Attachments
PT33, S4, G2 - Birds in a Forest - ManhattanLSAT.pdf
(77.04 KiB) Downloaded 930 times
 
nazu.s.shaikh
Thanks Received: 0
Elle Woods
Elle Woods
 
Posts: 53
Joined: April 27th, 2010
 
 
trophy
First Responder
 

Re: PT33, S4, G2 Bird-watchers explore a forest

by nazu.s.shaikh Fri May 14, 2010 2:34 pm

For this game how is it that question 7 the answer is E.. what differentiates answer D from E ? And the same goes for question 8... what makes D correct and not E


One more thing : For question 11... if G is in the forest then W must be in the forest correct? So why is answer A correct and not B?
User avatar
 
bbirdwell
Thanks Received: 864
Atticus Finch
Atticus Finch
 
Posts: 803
Joined: April 16th, 2009
 
 
 

Re: PT33, S4, G2 Bird-watchers explore a forest

by bbirdwell Sun May 16, 2010 6:54 pm

Hi.

Have you downloaded the diagram in Noah's earlier post? It's extremely helpful.

7) The difference between D and E is the phrase "at least" and the phrase "at most."

If M and H are in, G and W are out. What's left? J and S. One of them must be in because they can never both be out. So at least ONE additional kind of bird must be in the forest. Two COULD, but only one MUST. So D is incorrect.

E is correct. There are six birds. Two are in, two are out. At MOST, two more birds (S and J) can be in.

8) Again, look at the diagram from Noah's post and follow the inferences. If J is out:
in: S
out: J

Therefore it must be false that both H and S are NOT in the forest ("neither/nor" = "both not"), because S is in.

E is ok. H and S could be the only birds in the forest, no problem.

Sounds like you're going the wrong way with the conditional arrow regarding S and J. The constraint should be understood to mean:
if NOT S --> J; the contrapositive being if NOT J --> S

Do you see?

11) No, that's not correct. If G OUT --> W OUT. The contrapositive would be if W IN --> G IN. Again, you cannot determine anything by starting with "G IN." These can only be read from left to right. So G being in does not affect W at all.
I host free online workshop/Q&A sessions called Zen and the Art of LSAT. You can find upcoming dates here: http://www.manhattanlsat.com/zen-and-the-art.cfm
 
nazu.s.shaikh
Thanks Received: 0
Elle Woods
Elle Woods
 
Posts: 53
Joined: April 27th, 2010
 
 
trophy
First Responder
 

Re: PT33, S4, G2 Bird-watchers explore a forest

by nazu.s.shaikh Sat Aug 14, 2010 3:10 pm

I understand why A is correct for Q10 , that was one of my inferences.

But the question ask "Which one of the following pairs of birds CANNOT be among those birds contained in the forest?"

Maybe I misinterpreted the question, but J&S cannot be both contained in the forest together? ... How is that answer incorrect?
User avatar
 
bbirdwell
Thanks Received: 864
Atticus Finch
Atticus Finch
 
Posts: 803
Joined: April 16th, 2009
 
 
 

Re: PT33, S4, G2 Bird-watchers explore a forest

by bbirdwell Sat Aug 14, 2010 4:08 pm

Please refer to the previous post. S and J can be IN together, they just cannot be OUT together.

~J-->S
~S-->J

Read only from left to right! If one of the two is IN, you know NOTHING about the other...
I host free online workshop/Q&A sessions called Zen and the Art of LSAT. You can find upcoming dates here: http://www.manhattanlsat.com/zen-and-the-art.cfm
 
nazu.s.shaikh
Thanks Received: 0
Elle Woods
Elle Woods
 
Posts: 53
Joined: April 27th, 2010
 
 
trophy
First Responder
 

Re: PT33, S4, G2 Bird-watchers explore a forest

by nazu.s.shaikh Sat Aug 14, 2010 4:45 pm

Okay, let me see if I understand this correctly. ( Sorry I tend to over analyze everything and then manage to confuse myself of even the simplest things)

If J are not in the forest, S are.

If S are not in the forest, J ares.

Both J and S cannot be in the forest, at the same time... right? Or am I confusing the constraint.

I interpreted Q10 to be that which of the following pairs cannot be in the forest together?

So J and S can be together IN the forest, at the same time? ( Is that what you are saying?)
User avatar
 
bbirdwell
Thanks Received: 864
Atticus Finch
Atticus Finch
 
Posts: 803
Joined: April 16th, 2009
 
 
 

Re: PT33, S4, G2 Bird-watchers explore a forest

by bbirdwell Sat Aug 14, 2010 5:12 pm

Yes, you are right about the question: Which two birds can never be IN together?

In symbolizing the constraint regarding S and J, please look at my two previous posts, especially the part about reading the constraint from left to right. If J is in, what does that tell you about S? Nothing! If S is in, what does that tell you about J? Nothing!

Thus, it's ok for them to be IN together, at the same time, and that's why B isn't the answer.

It's not ok for both of them to be OUT together, as you can see by reading the constraint (if one is out, the other is in).
I host free online workshop/Q&A sessions called Zen and the Art of LSAT. You can find upcoming dates here: http://www.manhattanlsat.com/zen-and-the-art.cfm
 
jh2352
Thanks Received: 0
Forum Guests
 
Posts: 13
Joined: July 24th, 2013
 
 
 

Re: Diagram

by jh2352 Wed Sep 10, 2014 2:07 pm

Is rule 2, "If jays, martins or both are in the forest, then so are harriers", a compound conditional rule that should be written on the side?